Prove the Contraction Mapping Theorem.

Click For Summary
The Contraction Mapping Theorem states that in a complete metric space (X,d), a map g : X → X satisfying d(g(x), g(y)) ≤ λ d(x,y) for 0 < λ < 1 has a unique fixed point x* in X, attracting all sequences of iterates from any starting point x0. To prove this, one must show that the sequence generated by iterating g is Cauchy, which can be established by bounding the distances between terms using the geometric series. The discussion highlights the importance of correctly applying the contraction condition to derive bounds rather than assuming terms are zero. Clarification is sought on how to ensure the sequence converges, emphasizing the need for rigorous argumentation in mathematical proofs. The conversation ultimately revolves around the correct application of contraction properties to demonstrate convergence and uniqueness.
a358ask
Messages
3
Reaction score
0
Prove the Contraction Mapping Theorem.

Let ##(X,d)## be a complete metric space and ##g : X \rightarrow X## be a map such that ##\forall x,y \in X, d(g(x), g(y)) \le \lambda d(x,y)## for some ##0<\lambda < 1##.Then ##g## has a unique fixed point ##x^* \in X ##, and it attracts everything, i.e. for any ##x_0 \in X## , the sequence of iterates ##x_0, g(x_0), g(g(x_0))##, ... converges to the fixed point ##x^* \in X##.

The hint I am given are for existence and convergence - prove that the sequence is Cauchy. For uniqueness, choose two fixed points of ##g## and apply the map to both.

My approach is After we got to ##d(x_{n+1}, x_n) \le \lambda^n d(x_1, x_0)##,

then assuming that ##m > n##, ##d(x_m, x_n) \le d(x_m, x_{m-1}) + d(x_{m-1}, x_{m-2}) + ... + d(x_{n+1}, x_{n})## since each term of the right hand size is 0 ...so if we add up all the 0 terms, we get 0 on the right hand size. Therefore, ##d(x_m, x_n)## is 0. If this is right, then the question I have here is how do I guarantee that ##x_{m-1} > x_n##?
 
Physics news on Phys.org
a358ask said:
Prove the Contraction Mapping Theorem.

Let ##(X,d)## be a complete metric space and ##g : X \rightarrow X## be a map such that ##\forall x,y \in X, d(g(x), g(y)) \le \lambda d(x,y)## for some ##0<\lambda < 1##.Then ##g## has a unique fixed point ##x^* \in X ##, and it attracts everything, i.e. for any ##x_0 \in X## , the sequence of iterates ##x_0, g(x_0), g(g(x_0))##, ... converges to the fixed point ##x^* \in X##.

The hint I am given are for existence and convergence - prove that the sequence is Cauchy. For uniqueness, choose two fixed points of ##g## and apply the map to both.

My approach is After we got to ##d(x_{n+1}, x_n) \le \lambda^n d(x_1, x_0)##,

then assuming that ##m > n##, ##d(x_m, x_n) \le d(x_m, x_{m-1}) + d(x_{m-1}, x_{m-2}) + ... + d(x_{n+1}, x_{n})## since each term of the right hand size is 0 ...so if we add up all the 0 terms, we get 0 on the right hand size. Therefore, ##d(x_m, x_n)## is 0. If this is right, then the question I have here is how do I guarantee that ##x_{m-1} > x_n##?

Assuming you are trying to prove the sequence is Cauchy and that ##x_n=g^n(x_0)##, why do you think the terms on the right side are zero??
 
Dick said:
Assuming you are trying to prove the sequence is Cauchy and that ##x_n=g^n(x_0)##, why do you think the terms on the right side are zero??

I realize my reasoning a little bit off since the difference between m and n could be infinity , is any difference way to argue that the sum of the right hand side are zeros?
 
a358ask said:
I realize my reasoning a little bit off since the difference between m and n could be infinity , is any difference way to argue that the sum of the right hand side are zeros?

They aren't zeros. Try to bound it with a geometric series.
 
Dick said:
They aren't zeros. Try to bound it with a geometric series.

Use geometric in the right hand side and could you show a step or two for that?
 
a358ask said:
Use geometric in the right hand side and could you show a step or two for that?

You've got ##d(x_{n+1}, x_n) \le \lambda^n d(x_1, x_0)##. Use it!
 
Question: A clock's minute hand has length 4 and its hour hand has length 3. What is the distance between the tips at the moment when it is increasing most rapidly?(Putnam Exam Question) Answer: Making assumption that both the hands moves at constant angular velocities, the answer is ## \sqrt{7} .## But don't you think this assumption is somewhat doubtful and wrong?

Similar threads

  • · Replies 12 ·
Replies
12
Views
2K
  • · Replies 12 ·
Replies
12
Views
2K
  • · Replies 16 ·
Replies
16
Views
1K
  • · Replies 7 ·
Replies
7
Views
2K
Replies
10
Views
2K
Replies
5
Views
2K
Replies
2
Views
2K
  • · Replies 17 ·
Replies
17
Views
1K
  • · Replies 14 ·
Replies
14
Views
2K
  • · Replies 0 ·
Replies
0
Views
363